Difference between revisions of "2004 AMC 10A Problems/Problem 21"
Solzonmars (talk | contribs) (→Problem) |
Pi is 3.14 (talk | contribs) (→Solution 2) |
||
(12 intermediate revisions by 6 users not shown) | |||
Line 2: | Line 2: | ||
Two distinct lines pass through the center of three concentric circles of radii 3, 2, and 1. The area of the shaded region in the diagram is <math>\frac{8}{13}</math> of the area of the unshaded region. What is the radian measure of the acute angle formed by the two lines? (Note: <math>\pi</math> radians is <math>180</math> degrees.) | Two distinct lines pass through the center of three concentric circles of radii 3, 2, and 1. The area of the shaded region in the diagram is <math>\frac{8}{13}</math> of the area of the unshaded region. What is the radian measure of the acute angle formed by the two lines? (Note: <math>\pi</math> radians is <math>180</math> degrees.) | ||
− | <center> | + | <center><asy> |
+ | size(85); | ||
+ | fill((-30,0)..(-24,18)--(0,0)--(-24,-18)..cycle,gray(0.7)); | ||
+ | fill((30,0)..(24,18)--(0,0)--(24,-18)..cycle,gray(0.7)); | ||
+ | fill((-20,0)..(0,20)--(0,-20)..cycle,white); | ||
+ | fill((20,0)..(0,20)--(0,-20)..cycle,white); | ||
+ | fill((0,20)..(-16,12)--(0,0)--(16,12)..cycle,gray(0.7)); | ||
+ | fill((0,-20)..(-16,-12)--(0,0)--(16,-12)..cycle,gray(0.7)); | ||
+ | fill((0,10)..(-10,0)--(10,0)..cycle,white); | ||
+ | fill((0,-10)..(-10,0)--(10,0)..cycle,white); | ||
+ | fill((-10,0)..(-8,6)--(0,0)--(-8,-6)..cycle,gray(0.7)); | ||
+ | fill((10,0)..(8,6)--(0,0)--(8,-6)..cycle,gray(0.7)); | ||
+ | draw(Circle((0,0),10),linewidth(0.7)); | ||
+ | draw(Circle((0,0),20),linewidth(0.7)); | ||
+ | draw(Circle((0,0),30),linewidth(0.7)); | ||
+ | draw((-28,-21)--(28,21),linewidth(0.7)); | ||
+ | draw((-28,21)--(28,-21),linewidth(0.7));</asy></center> | ||
<math> \mathrm{(A) \ } \frac{\pi}{8} \qquad \mathrm{(B) \ } \frac{\pi}{7} \qquad \mathrm{(C) \ } \frac{\pi}{6} \qquad \mathrm{(D) \ } \frac{\pi}{5} \qquad \mathrm{(E) \ } \frac{\pi}{4} </math> | <math> \mathrm{(A) \ } \frac{\pi}{8} \qquad \mathrm{(B) \ } \frac{\pi}{7} \qquad \mathrm{(C) \ } \frac{\pi}{6} \qquad \mathrm{(D) \ } \frac{\pi}{5} \qquad \mathrm{(E) \ } \frac{\pi}{4} </math> | ||
− | ==Solution== | + | ==Solution 1== |
Let the area of the shaded region be <math>S</math>, the area of the unshaded region be <math>U</math>, and the acute angle that is formed by the two lines be <math>\theta</math>. We can set up two equations between <math>S</math> and <math>U</math>: | Let the area of the shaded region be <math>S</math>, the area of the unshaded region be <math>U</math>, and the acute angle that is formed by the two lines be <math>\theta</math>. We can set up two equations between <math>S</math> and <math>U</math>: | ||
Line 20: | Line 36: | ||
Thus <math>3\theta=\dfrac{3\pi}{7}\Rightarrow \theta=\dfrac{\pi}{7}\Rightarrow\boxed{\mathrm{(B)}\ \frac{\pi}{7}}</math> | Thus <math>3\theta=\dfrac{3\pi}{7}\Rightarrow \theta=\dfrac{\pi}{7}\Rightarrow\boxed{\mathrm{(B)}\ \frac{\pi}{7}}</math> | ||
+ | |||
+ | ==Solution 2== | ||
+ | |||
+ | As mentioned in Solution #1, we can make an equation for the area of the shaded region in terms of <math>\theta</math>. | ||
+ | |||
+ | <math>\implies\dfrac{2\theta}{2\pi} \cdot \pi +\dfrac{2(\pi-\theta)}{2\pi} \cdot (4\pi-\pi)+\dfrac{2\theta}{2\pi}(9\pi-4\pi)=\theta +3\pi-3\theta+5\theta=3\theta+3\pi</math>. | ||
+ | |||
+ | So, the shaded region is <math>3\theta+3\pi</math>. This means that the unshaded region is <math>9\pi-(3\theta+3\pi)</math>. | ||
+ | |||
+ | Also, the shaded region is <math>\frac{8}{13}</math> of the unshaded region. Hence, we can now make an equation and solve for <math>\theta</math>. | ||
+ | |||
+ | |||
+ | <math>3\theta+3\pi=\frac{8}{13}(9\pi-(3\theta+3\pi)\implies 39\theta+39\pi=8(6\pi-3\theta)\implies 39\theta+39\pi=48\pi-24\theta</math>. | ||
+ | |||
+ | Simplifying, we get <math>63\theta=9\pi\implies \theta=\boxed{\mathrm{(B)}\ \frac{\pi}{7}}</math> | ||
+ | |||
+ | == Video Solution by OmegaLearn == | ||
+ | https://youtu.be/t3EWtMnJu2Y?t=49 | ||
+ | |||
+ | ~ pi_is_3.14 | ||
== See also == | == See also == |
Latest revision as of 03:50, 23 January 2023
Problem
Two distinct lines pass through the center of three concentric circles of radii 3, 2, and 1. The area of the shaded region in the diagram is of the area of the unshaded region. What is the radian measure of the acute angle formed by the two lines? (Note: radians is degrees.)
Solution 1
Let the area of the shaded region be , the area of the unshaded region be , and the acute angle that is formed by the two lines be . We can set up two equations between and :
Thus , and , and thus .
Now we can make a formula for the area of the shaded region in terms of :
Thus
Solution 2
As mentioned in Solution #1, we can make an equation for the area of the shaded region in terms of .
.
So, the shaded region is . This means that the unshaded region is .
Also, the shaded region is of the unshaded region. Hence, we can now make an equation and solve for .
.
Simplifying, we get
Video Solution by OmegaLearn
https://youtu.be/t3EWtMnJu2Y?t=49
~ pi_is_3.14
See also
2004 AMC 10A (Problems • Answer Key • Resources) | ||
Preceded by Problem 20 |
Followed by Problem 22 | |
1 • 2 • 3 • 4 • 5 • 6 • 7 • 8 • 9 • 10 • 11 • 12 • 13 • 14 • 15 • 16 • 17 • 18 • 19 • 20 • 21 • 22 • 23 • 24 • 25 | ||
All AMC 10 Problems and Solutions |
The problems on this page are copyrighted by the Mathematical Association of America's American Mathematics Competitions.